Which word describes Arette?
A. Trilingual
B. Bilingual
C. Monolingual

Answers

Answer 1

Arette is considered bilingual.

Within the context of this question, Arette is from France but speaks two different language

What is bilingual?

Bilingual is a word which is usually, frequently and most of the times used to describes a person, community or a nation that speaks two languages.

I myself too; I am bilingual; I speak English and Yoruba

Generally, people who speak more more more than one language are considered as linguists

In conclusion, Arette is considered bilingual.

Learn more about linguists:

https://brainly.com/question/904593

#SPJ1


Related Questions

A ball is launched from the ground with a horizontal speed of 30 m/s and a vertical speed of 30 m/s. How long will it take to get to the top of its trajectory?

Answers

The ball will take 3 seconds to get to the top of its trajectory When it is launched from the ground with a horizontal speed of 30 m/s and a vertical speed of 30 m/s

What is Trajectory ?

When any object is thrown from horizontal at an angle θ except 90°, then the path followed by it is called trajectory, the object is called projectile and its motion is called projectile motion.

Here, we have a type of motion called projectile motion and it is pretty similar to an upside down parabola. The top of the trajectory is the vertex of the parabola and is also when v = 0.

Given :

Horizontal speed= 30m/sVertical Speed= 30 m/s

This problem is now relatively easy because we only need to find the vertical distance so we can ignore horizontal speed and use

vy = vy0 + ayt

Plug in our givens

0 = 30  - 10t

solve for t

t = 3 seconds

Hence, The ball will take 3 seconds to get to the top of its trajectory When it is launched from the ground with a horizontal speed of 30 m/s and a vertical speed of 30 m/s

Learn more about Projectile motion here ;

https://brainly.com/question/11049671

#SPJ1

If you place a 3.5-kg object on a spring which has a spring constant 144-N/m, and stretch the spring so the object starts oscillating. How much time does it take to complete one cycle

Answers

Answer:

M = 3.5 kg         mass of object

F = - K x     where K is the spring constant

K = 144 N/m

ω = (K/M)^1/2     for simple harmonic motiom

ω = (144 kg/s^2 / 3.5 kg)^1/2 = 6.41 / sec

f =  ω / (2 * π)  = 1 / P

P = (2 Π / ω) = 2 * 3.14 / 6.41 sec

P = .98 sec

Check: f = 1 / P = 1.02 /sec

ω = 2 * π * f = 2 * 3.14 * 1.02 = 6.41 sec

In which of the following scenarios is the total momentum of the system conserved?

Answers

Answer:

The total momentum of a system is conserved only when the system is closed.

Explanation:

You look in the sky and see two jetliners that you know are of equal size, yet one appears to be much larger. Because of your knowledge of __________, you will assume that the smaller jetliner is farther away

Answers

Answer:

Because of the knowledge of relative size, it will be assumed that the smaller jetliner is farther away.

Explanation:

According to the theory of relative size, the distance that an object has to the viewing individual affects the perception of the individual regarding the size of the object.

As stated in this case, one of the jetliners is farther away from the other. Therefore, even if the jets are of equal size, the one that is at a greater distance is perceived to be smaller as it is at a greater viewing range. The one that is nearer to the individual seems bigger in comparison to the one farther away due to a closer viewing range.

Therefore, the jet that is nearer appears larger.

To know more about relative size, refer to:

https://brainly.com/question/19998265

#SPJ4

At which points on the roller coaster is the car not moving?

A
A and E

B
B and C

C
C and D

D
D and E

Answers

The maximum potential energy of the car or the when car is not moving at points A and E; option A.

What is potential energy?

Potential energy is the energy of a body due to its state or position.

In the rollercoaster motion shown, the maximum potential energy occurs when the car is no longer moving.

At different points other than at maximum potential energy, the energy is a combination of potential and kinetic energy.

The maximum potential energy occurs at A and E, at which point the car is not moving.

In conclusion, at maximum potential energy, the car is not moving.

Learn more about potential energy at: https://brainly.com/question/14427111

#SPJ1

The density of a certain type of plastic is 0.77 g/cm3. If a sheet of this plastic is 10.0 m long, 1.0 m wide, and 1 cm thick, what is its mass

Answers

Mass is 77,000 g

Given

The density of a certain type of plastic = 0.77 [tex]$\mathrm{g} / \mathrm{cm}^{3}$[/tex]

Length = 10.0 m = 1000 cm

Width = 1 m = 100 cm

Thickness = 1 cm

Formula

Volume = length x width x thickness

Mass = Density x Volume

Explanation

Density

Density is how much "stuff" is contained in a specific quantity of space is determined by its density. For instance, a block of the harder, lighter element gold (Au) will be denser than a block of the heavier element lead (Pb) (Au). Styrofoam blocks are less dense than bricks. Mass per unit volume serves as its definition.

Volume V = 1000 x 100 x 1

             V = 100,000 [tex]cm^{2}[/tex]

Mass M= 0.77 [tex]$\mathrm{g} / \mathrm{cm}^{3}$[/tex] x 100,000 [tex]cm^{2}[/tex]

M = 77,000 g

So, Mass is 77,000 g

Learn more about this problem here:

https://brainly.com/question/598902

#SPJ4

If f is the magnitude of the force on the test charge due to only one of the other charges, what is the magnitude of the net force acting on the test charge due to both of these charges?.

Answers

The net force acting on test charge is resultant of two force i.e.,  √2 F

The magnitude of force on test charge due to one charge Q is:

                F = k qQ / r²

where k = 9 × 10^9

Similarly the force due to second charge will be:

         F = k qQ / r²

Now both the charges of magnitude Q is present and it is pointed at right angle to each other then the force  will be resultant i.e.,

                    F = √ F² + F² = F√2

 Hence, net force acting on test charge is √2 F.

  Learn more about charges here:

        https://brainly.com/question/15840431

             #SPJ4

A 500 N weight is hung at the middle of a rope attached to two buildings at the same level. If the breaks in the tension exceed 1800 N, what is the minimum angle the rope can make with the horizontal? Group of answer choices 4o 8o 11o 18o

Answers

Answer:

The correct choice is 8o.

Explanation:

The given weight is hanging at the mid point of the rope and the buildings are at the same level, Obtain the given equation by equating the vertical components of force,

2Tsin(φ)=W

where φ is the angle made by rope with the horizontal.

Given T=1800 N and W=500 N, the value of φ is calculated as follows,

2*1800*sin(φ)=500

sin(φ)=0.1388

φ=7.90

φ≈8o

Check out similar questions.

brainly.com/question/13735944

#SPJ10

4. An applied force of 6.2 N acts on a 2.1-kg object, pushing it horizontally across a surface
where the coefficient of kinetic friction is 0.15.
a. Draw a free body diagram for the body
b. Determine the frictional force acting.
c. Determine the net force acting on the body
d. Determine the object's acceleration.
Step by step, thank you
[2C]
[2A]
[2A]
[2A]

Answers

The frictional force on the body is  0.25 N the net force on the body is  5.95 N while the acceleration produced is 2.83 m/s^2

What is the net force?

The net force is used to describe the resultant force that acts on the object.

The frictional force that acts on the object s obtained from;

Ff = 0.15 * 2.1-kg * 9.8 m/s^2

Ff = 0.25 N

The net force that acts on the body = F - Ff where F is the applied the force

=  6.2 N - 0.25 N

Net force = 5.95 N

Now;

Net force = ma

a= Net force /m

a = 5.95 N/2.1-kg

a = 2.83 m/s^2

Learn more about the net force:https://brainly.com/question/18031889

#SPJ1

A brightfield microscope equipped with a ____________ ___________ stop is able to block light from the illuminator creating bright objects on a dark background.

Answers

Answer: A Brightfield microscope equipped with an opaque light stop is able to block light from the illuminator and creating bright objects on a dark background.

Explanation: To find the correct statement about Brightfield microscope, we need to know more about the Brightfield microscope.

what is Brightfield microscope?The Brightfield microscopic technique is generally used with the compound microscopes and, these are one, among the widely used optical illumination techniques. The Brightfield microscope or Compound light microscope is an optical instrument, which uses light rays and produces dark images with a bright background.How Bright field microscope works?A brightfield microscope equipped with a opaque light stop, will produce bright objects on a dark background by blocking the light illuminated from the light source. This opaque light stop, blocks the light rays that arising from the illuminator to the objective lenses and, only allows the reflected or transmitted light of the specimen. As a result, we get bright objects with a dark background.

Thus, we can conclude that, A Brightfield microscope equipped with an opaque light stop is able to block light from the illuminator and creating bright objects on a dark background.

Learn more about the Brightfield microscope here: https://brainly.com/question/28019837

#SPJ4

Without using afood flask, suggest another ways would you keep food warm a till to the hospital

Answers

Another way to keep food warm till you arrive at the hospital is by putting the food in a foil and wrapping it up with a towel to retain the heat.

What is Food preservation?

Food preservation is defined as the process by which a cooked food or process food is kept in such a way that microorganisms cannot affect their taste and texture.

To prevent a cooked food from getting cold, you can put the food in an aluminium foil and wrap it with a towel to prevent heat loss.

Learn more about food preservation here:

https://brainly.com/question/16972821

#SPJ1

An aircraft executes a horizontal loop of radius 1.00 km with a steady speed of 900 km/h.Compare it's centripetal acceleration with the acceleration due to gravity.​

Answers

Answer:

900 km/hr = 9E5 m / 3600 sec = 250 m/sec

ah = v^2 / R = 250^2 m/s / 1000 m = 62.5 m/s^2   horizontal acceleration

av = 9.8 m/s^2      vertical acceleration (due to gravity)

ah / av = 62.5 / 9.8 = 6.38   centripetal acceleration is greater

a ray of light incident on a mirror, at an angle of 45°. Another mirror is placed at an angle of 45° to the first ones as shown. Sketch the patch of the ray until it emerges from the two mirrors (2mks) 45° www.www 45° ​

Answers

Answer:

If the ray of light is deflected by 45 degrees by the first mirror its total deflection by mirror (I) is 90 deg. (incident = 45 and exit ray equals 45 deg)

The second mirror will cause a net deflection of 90 degrees and the total deflection will be 180 deg or in opposite  direction to the  incident ray.

What is the current flowing through the circuit shown? (V= 120 V, R₁ = 20 02,
R₂= 50 Q, R3= 1022) (Ohm's law: V = IR)
R₁
ww
ww
R₂
ww
R₂

Answers

Assumed that resistors are connected in series .

R_net=20+50+10=80ohm

Voltage=V=120V

Current=I

V/R120/803/21.5A


Which statement is true of a piece of ice at 0°C that is put into a freezer at
-18°C?

Answers

The statement which is true about a piece of ice at 0°C which is put into a freezer at -18°C is it having the temperature of the freezer.

What is Temperature?

This is referred to the degree of hotness or coldness of a body and the unit is Celsius or Kelvin.

The ice at 0°C will experience a change in temperature of the freezer when put in it in this scenario.

Read more about Ice here https://brainly.com/question/2267329

#SPJ1

Answer the following questions.
1. What is the velocity of a car that traveled a total of 75 kilometers north in 1.5 hours?
2. What is the velocity of a plane that traveled 3,000 miles from New York to California in 5.0 hours?
3. John took 45 minutes to bicycle to his grandmother's house, a total of four kilometers. What was his speed in km/hr?.
4. It took 3.5 hours for a train to travel the distance between two cities at a speed of 120 miles/hr. How many miles lie between the two cities?​

Answers

3.if time=45mins,D=4km,2700secs,4000m then. S=d/t S=1.4m/s

A speed skater moving across frictionless ice at 8.0 m/s hits a 5.0-m-wide patch of rough ice. She slows steadily, then continues on at 6.0 m/s. What is her acceleration on the rough patch

Answers

Her acceleration on the rough patch is 2.8m/s².

To find the answer, we need to know about the Newton's equation of motion.

What are the Newton's equation of motion?

These are as follows

V= U +atS = Ut+1/2 at²V²-U²= 2aS

V= Final velocity

U = initial velocity

t= time

S= distance

What will be acceleration if initial velocity, final velocity and distance are 8 m/s , 6 m/s and 5m respectively?Here U= 8 m/s, V = 6 m/s and S = 5mSo, 6²-8²= 2a× 5=10a

36-64 = 10a

-28= 10 a

a= -28/10= -2.8 m/s²

Negative sign indicates the decrease of acceleration.

Thus, we can conclude that the acceleration on the patch is 2.8 m/s².

Learn more about the Newton's equation of motion here:

brainly.com/question/8898885

#SPJ4

While standing at the top of an 100 m high observatory, you accidentally dropped your phone through the grates (it is now falling straight down to the ground). a. What is the velocity of the phone after 4.0 s? b. How far does the phone fall during this time? C. Will your phone hit the ground? If not, how long more before it hits the ground?​

Answers

Using the idea of motion under gravity, it will take 0.5 seconds more before the phone hits the ground.

What is the distance covered?

The distance covered is obtained form the equations of kinematics under gravity.

Now;

v = u + gt

Recall that it was dropped from a height hence u = 0 m/s

v = gt

v = 9.8 m/s^2 * 4 s

v = 39.2 m/s

Now;

h = ut + 1/2gt^2

but u = 0 m/s

h = 1/2gt^2

h = 1/2 * 9.8 * (4)^2

h = 78.4 m

The phone  will not hit the ground within this time

For the phone to hit the ground;

h = 1/2gt^2

if h = 100 m

100 = 1/2 * 9.8 * t^2

2 * 100/9.8 =  t^2

t = √2 * 100/9.8

t = 4.5 seconds

It will take about 0.5 seconds more before the phone will hit the ground.

Learn more about motion under gravity:https://brainly.com/question/15120445

#SPJ1

Calculate the frequency of the 3rd harmonic of a 0.5 meter string with a linear density of 0.03 kg/m at a tension of 50 newtons.

Answers

Frequency of third harmonic is 122.48 HZ

Theory:

Frequency of third harmonic with length L , linear density μ , Tension

T is given as

                  fₙ= n v/2L

                   fₙ = n/2L × √T/√μ

      plugging in the values here-

                   fₙ =  3/ 2×0.5 × √50 / √0.03

                   fₙ = 122.48  Hz

          Thus, the frequency of third harmonic is 122.48 HZ

        Learn more about frequency here:

               https://brainly.com/question/14888403

                      #SPJ4    

Two objects, m1 = 0.6 kg and m2 = 4.4 kg undergo a one-dimensional head-on collision
Their initial velocities along the one-dimension path are vi1 = 32.4 m/s [right] and vi2 = 8.6 m/s [left].
The two objects stick together after the perfectly inelastic collision
a) Calculate the velocity after the collision.
b) Determine how much kinetic energy is lost due to the collision

Answers

Explanation:

Hello !

look at the attachment above ☝️ and if you have any questions your welcome.

a) The velocity after the collision.is 11.456 m/s.

b) The kinetic energy lost due to the collision is 44.564 J.

What is conservation of momentum principle?

When two bodies of different masses move together each other and have head on collision, they travel to same or different direction after collision.

The external force is not acting here, so the initial momentum is equal to the final momentum. For inelastic collision, final velocity is the common velocity for both the bodies.

m₁u₁ +m₂u₂ =(m₁ +m₂) v

Given are the two objects, m1 = 0.6 kg and m2 = 4.4 kg undergo a one-dimensional head-on collision. Their initial velocities along the one-dimension path are vi1 = 32.4 m/s [right] and vi2 = 8.6 m/s [left].

(a) Substitute the values, then the final velocity will be

0.6 x32.4 +4.4 x 8.6 = (0.6+4.4)v

v = 11.456 m/s

Thus, the velocity after collision is 11.456 m/s.

(b) Kinetic energy lost due to collision will be the difference between the kinetic energy before and after collision.

= [1/2m₁u₁² +1/2m₂u₂² ] - [1/2(m₁ +m₂) v²]

Substitute the value, we have

= [1/2 x 0.6 x32.4² + 1/2 x4.4 x 8.6²] - [1/2 x(0.6+4.4)11.456²]

= 44.564 J

Thus, the kinetic energy lost due to the collision is 44.564 J.

Learn more about conservation of momentum principle

https://brainly.com/question/14033058

#SPJ2

If an electron has an uncertainty in its velocity of 1.40 m/s, what is the uncertainty (in meters) in its position

Answers

Uncertainty in position:

If the electron has an uncertainty in its velocity of 1.40 m/s then the uncertainty in its position is [tex]4.14\times10^{-4} \text{ m}[/tex].

Heisenberg's uncertainty principle to calculate the required:

Step-1:

We have to apply Heisenberg's uncertainty principle to calculate the uncertainty in the position of an electron. According to the principle:

[tex]$\Delta x \Delta p \geq \frac{h}{4 \pi}$[/tex]

Here,

[tex]$\Delta x$[/tex] is the uncertainty on the position measurement

[tex]$\Delta p$[/tex] is the uncertainty on the momentum measurement

h is the Planck constant.

It is known that the momentum of a particle is calculated as, the product of the mass of the particle and its velocity.

Therefore,

p=mv

Thus the Heisenberg principle becomes:

[tex]$m \Delta x \Delta v \geq \frac{h}{4 \pi}$[/tex]

Here [tex]$\Delta v$[/tex] is the uncertainty in the velocity measurement.

It is given that, [tex]$\Delta v$[/tex]=1.40 m/s. The particle is electron here, thus the mass of the particle m=[tex]9.1 \cdot 10^{-31} \mathrm{~kg}[/tex] and the value of the Plank's constant is, h=[tex]6.62 \times 10^{-34} \mathrm{~m}^{2} \mathrm{~kg} / \mathrm{s}[/tex]

Step-2:

Substituting the values into the equation to get the value of the position uncertainty.

[tex]\Delta x \geq \frac{h}{4 \pi\times m\times \Delta v}\\\geq\frac{6.62\times10^{-34}}{4\pi \times 9.1\times10^{-31}\times1.40} \text{ m}\\\geq 4.14\times10^{-4} \text{ m}[/tex]

To know more about the uncertainty in position, refer to:

https://brainly.com/question/9574825

#SPJ4

A glass optical fiber is used to transport a light ray across a long distance. The fiber has an index of refraction of 1.510 and is submerged in water, which has an index of refraction of 1.333. What is the critical angle (in degrees) for the light ray to remain inside the fiber

Answers

The answer is 73.13°.

According to snell's law,

n1sinθi = n2sinθr

n1/n2 = sinθr/sinθi

The critical angle is the angle of incidence at the denser medium when the angle of incidence at the less dense medium is 90°

This means i=C and r = 90°

The Snell's law formula will become

n1/n2 = sinC/sin90°

n2/n1 = 1/sinC

Where n1 is the refractive index of the less dense medium = 1.473

n2 is the refractive index of the denser medium = 1.540

Substituting the values in the formula,

1.540/1.473 = 1/sinC

1.045 = 1/sinC

SinC = 1/1.045

SinC = 0.957

C = sin^-1(0.957)

C = 73.13°

The refractive index of glass is 1.5. It way that the velocity of mild in glass is 1.5 times slower than the rate of mild in a vacuum, the speed of light in glass isn't always unbiased of the shade of mild.

Refractive index is likewise the same as the velocity of light c of a given wavelength in an empty area divided with the aid of its speed v in a substance or n = c/v.

The index of refraction of fabric is a ratio that compares the velocity of light in a vacuum ( c=3.00x108ms ) to the velocity of mild in that precise medium. Because the index of refraction increases, the amount that the material bends the mild increases.

Learn more about the index of refraction here https://brainly.com/question/12469161

#SPJ4

The critical angle (in degrees) for the light ray to remain inside the fiber is 73.13°.

According to snell's law,

n1sinθi = n2sinθr

n1/n2 = sinθr/sinθi

The critical angle is the angle of incidence at the denser medium when the angle of incidence at the less dense medium is 90°.

This means i=C and r = 90°

The Snell's law formula will become

n1/n2 = sinC/sin90°

n2/n1 = 1/sinC

Where n1 is the refractive index of the less dense medium = 1.473

n2 is the refractive index of the denser medium = 1.540

Substituting the values in the formula,

1.540/1.473 = 1/sinC

1.045 = 1/sinC

SinC = 1/1.045

SinC = 0.957

C = sin^-1(0.957)

C = 73.13°

Learn more about critical angle here https://brainly.com/question/15009181

#SPJ4

Throw two balls from the same height at the same time at an initial speed of 20 m/s. One is thrown vertically down, while the other is thrown vertically up. We want to find the time difference between their landing.

Answers

The time difference between their landing is 2.04 seconds.

Time of difference of the two balls

The ball thrown vertical upwards will take double of the time taken by the ball thrown vertically downwards.

Time difference, = 2t - t = t

t = √(2h/g)

where;

h is the height of fallg is acceleration due to gravity

Apply the principle of conservation of energy;

¹/₂mv² = mgh

h = v²/2g

where;

v is speed of the ball

h = (20²)/(2 x 9.8)

h = 20.41 m

Time of motion

t = √(2 x 20.41 / 9.8)

t = 2.04 s

Thus, the time difference between their landing is 2.04 seconds.

Learn more about time of motion here: https://brainly.com/question/2364404

#SPJ1

If the radio waves transmitted by a radio station have a frequency of 79.5 MHz, what is the wavelength of the waves, in meters

Answers

If the radio waves transmitted by a radio station have a frequency of 79.5 MHz, the wavelength of the waves, in meters is 3.77m.

What are radio waves?The electromagnetic spectrum's longest wavelengths, which are found in radio waves, are normally found at frequencies of 300 gigahertz. They may be reflected and refracted to change direction, and they do not harm the human body if they are absorbed by it. They are perfect for communicating due to their characteristics.

What are Frequency and wavelength?When describing the temporal rate of change seen in oscillatory and periodic phenomena like mechanical vibrations, radio waves, and light, frequency is a crucial parameter utilized in science and engineering.The length of a waveform signal that is propagating in space or over a wire is measured by the separation between identical points (adjacent crests) in the adjacent cycles. Frequency, or the number of wave cycles per second, is inversely related to wavelength.

Given: Velocity of light,  c = 3.00 x 10⁸ m/s  m/s

Frequency, f = 7.95 x 10⁷/s

For wavelength, the required formula is

[tex]λ = c/ f \\λ = 3 * 10 ^{8} / 7.95 * 10^{7} \\λ = 3.77[/tex]

Hence, the required wavelength is 3.77 meters.

To learn more about frequency and wavelength refer to:

https://brainly.com/question/12924624

#SPJ4

_____________is the addition of wave energy as waves interact, producing larger waves and ____________ is the subtraction of wave energy as waves interact, producing smaller waves.

Answers

Constructive interference is the addition of wave energy as waves interact, producing larger waves and destructive interference is the subtraction of wave energy as waves interact, producing smaller waves.

To find the answer, we need to know about the interference of waves.

What's the interference of waves?Interference of waves is the result of superposition of waves (transverse or longitudinal) at a certain place. Interference is of two typesConstructive interferenceConstructive interferenceDestructive interferenceHow are the constructive and destructive interference formed?When two waves are superimposed within the phase, then constructive interference pattern is formed.When two waves are superimposed with out of phase, destructive interference pattern is found.

Thus, we can conclude that the constructive interference is the addition of wave energy and destructive interference is the subtraction of wave energy.

Learn more about the interference of waves here:

brainly.com/question/12466679

#SPJ4

A block of mass 60kg measures 6cm by 5cm by 4cm. Calculate i) The maximum pressure it can exert. The minimum pressure. ii) A block of mass 60kg measures 6cm by 5cm by 4cm . Calculate i ) The maximum pressure it can exert . The minimum pressure . ii )​

Answers

Answer:

4.9 is the answer

Explanation:

pressure is force divided by area

force = mass x acceleration due to gravity

= 60kg x 9.8 m/s

area = length x width x height

=6 x 5 x 4

= 120

pressure = 588/120

= 4.9 pascal

The correct answer for the value of (i) Maximum pressure is [tex]24.5 \dfrac{N}{cm^2}[/tex](ii)Minimum pressure is [tex]19.6 \dfrac{N}{cm^2}[/tex].

Given:

Weight of the block:  [tex]60[/tex] kg

Dimensions of the box:  [tex]6 *4* 5[/tex]

Value of gravitational constant:  [tex]9.8[/tex] m/s²

Now,

[tex]Pressure = \dfrac{Force}{Area}[/tex]

[tex]Force = mass*accelaration[/tex]

[tex]= 60 * 9.8[/tex]

[tex]= 588 N[/tex]

Minimum pressure is when area is maximum:

[tex]= \dfrac{588}{6*5}\\\\= 19.6 \dfrac{N}{cm^2}[/tex]

Maximum pressure when area is minimum:

[tex]= \dfrac{588}{6*4} \\\\= 24.5 \dfrac{N}{cm^2}[/tex]

Maximum and minimum pressure are   [tex]24.5 \dfrac{N}{cm^2}[/tex]  and  [tex]19.6 \dfrac{N}{cm^2}[/tex] respectively.

Learn more about Pressure here:

https://brainly.com/question/21173163

#SPJ7

A vehicle starts from rest and accelerates uniformly for 12 seconds to a
velocity of 10ms1. It then runs at a constant velocity and is finally came to rest
in 66m with constant . The total distance covered by the vehicle is
580m. Find the value of acceleration and the time taken.

Answers

The value of the acceleration is 0.76 m/s² and the total time taken by the vehicle is 39 seconds.

Acceleration of the vehicle

The acceleration of the vehicle before coming to rest is calculated as follows;

v² = u² + 2as

where;

v is the final velocityu is the initial velocitya is the accelerations is the distance traveled before stopping

the car came to rest with constant velocity attained after 12 seconds.

the initial velocity of the car before 12 seconds is zero.

v² = 0 + 2as

a = v²/2s

a = (10²)/(2 x 66)

a = 0.76 m/s²

Time of motion of the vehicle

d = ut + ¹/₂at²

where;

d is the total distance traveledt is the time of motiona is accelerationu is initial velocity of the vehicle

580 = 0 + ¹/₂(0.76)t²

580 = 0.38t²

t² = 580/0.38

t² = 1,526.3

t = √1,526.3

t = 39 seconds

Thus, the value of the acceleration is 0.76 m/s² and the total time taken by the vehicle is 39 seconds.

Learn more about time of motion here: https://brainly.com/question/2364404

#SPJ1

. A man of mass 80 kg stands in a lift, what is the reaction from the floor of the lift if the lift;A. moves upwards with steady speed. moves upwards with acceleration 0.5m/sC. moves downward with acceleration 0.4m/s(g - 9.8m/s')​

Answers

The reaction of the floor is as follows:

R = 784 NR = 816 N

What is the reaction from the floor for a man in a lift?

The reaction of the floor on the man is equal to the weight of the man and the upward force.

Reaction, R = mg + ma

A)  When the lift moves upwards with steady speed.

R = 80 * 9.8 + * 80 * 0

R = 784 N

B) When the lift moves upwards with acceleration 0.4m/s

R = 80 * 9.8 + * 80 * 0.4

R = 816 N

In conclusion, the reaction of the floor changes as the body accelerates upwards.

Learn more about reaction and weight at: https://brainly.com/question/25555136

#SPJ1

(picture attached) use grass method
4. A block of unknown substance is submerged in water. A light ray in the water strikes the angle of 16°, what is the index of refraction of the unknown substance? (nwater = 1.33) substance at an angle of 45° from the normal. If the refracted ray in the substance is at an 5. If a light ray passes from a substance with low index of refraction to another substance with high index of refraction, will the ray bend away from or closer to the normal? Why? 6. What is the angle of refraction? n₁=1 24° n₂ = 1.8​

Answers

Answer:

Explanation:

Step 1

1 of 2

\textbf{Given}Given

n_{w}=1.33n

w

=1.33

$\theta_w=31\text{ ^\circ

}$

$\theta_b=27\text{ ^\circ

}$

\textbf{Approach}Approach

In this problem we are going to use Snell's law.

\textbf{Solution}Solution

The definition of Snell's law of refraction is

\begin{align} {n_1\cdot\sin \theta_1}={n_2\cdot \sin \theta_2} \end{align}

n

1

⋅sinθ

1

=n

2

⋅sinθ

2

 

where indexes 11 and 22 represent two different mediums. Since the motion is in the water we write

\begin{align} &{n_{w}\cdot\sin\theta_{w}}={n_{b}\cdot\sin\theta_{b}} \\ &{n_b}={n_{w}\cdot \frac{\sin\theta_{w}}{ \sin \theta_{b}} } \\ &{n_b}={1.33\cdot \frac{\sin31^\circ}{ \sin27^\circ} } \\ &\boxed{{n_b}=1.5} \end{align}

 

n

w

⋅sinθ

w

=n

b

⋅sinθ

b

n

b

=n

w

sinθ

b

sinθ

w

n

b

=1.33⋅

sin27

sin31

n

b

=1.5

Why should a scientist not be concerned when a fellow scientist responds to their conclusions with skepticism?

Answers

ABCDEDJGHIKLMNOP

just wanted to see if bold work
Other Questions
What role did religion play in Martin Luther King Jrs youth Which is bigger, a third of 45 or one-sixth c138? The ability of a nonpathogen or weakly pathogenic microorganism to cause disease primarily in an immunocompromised host is termed a(n) ______ infection. three ways that cultural diffusion can happen It is finals week at State University and Agnes has five finals in three days. What phase of the stress process is she probably experiencing if she just went through all of the kitchen cupboards searching for a package of chocolate bars loaded with caramel which she just remembered she hid before spring break A gas can hold 10 litters of gas.How many could we fill 7 litters of gas? Karn is a Hindu whose spiritual path involves renouncing the world. Karn has quit his job and lives as a sannyasin so that he can focus entirely on attaining moksha. He studies the Upanishads and hopes to escape samsara by fully realizing that atman is one with brahman. Which path does Karn most likely practice HELPMarbles: You have a bag with 3 red marbles, 4 blue marbles, and 5 green marbles. What is the probability of pulling out a red marble 3 times in a row, assuming you put each marble back into the bag before drawing another? What is judicial review?A.The power of the Judicial Branch to review laws, Executive orders, and Executive acts to ensure that they are Constitutional.B.The power of the Judicial Branch to impeach the President and members of Congress.C.The power of the Legislative Branch to appoint or remove Supreme Court Judges at any time.D.The power of the Executive Branch to deem Supreme Court rulings unconstitutional. HELP!! This is part of a big project I have and its a big part of my grade please help Cells can make new cells. One cell can (a) ____________ into two new cells. This is called (b)__________________. The process of cell division goes through various states. First, the cell nucleus (c)________________ into two. A new cell surface membrane then (d)____________ the cell divides. The two new cells are called (e)_______________ and they are small. They will grow and become larger. They grow by getting (f)______________ from the food that is eaten. Once they grow to full size they can also (g)_____________. If cells divide more quickly than they should, or divide in the wrong way, (h)_____________ can develop. What is the answer to this Identify the algebraic expression for the following: the quotient of 12 and a number y increased by 7 7. Tyrone is thinking of a number. In his number:- The digit 4 has a value of 0.04- The digit 3 is in both the ones and tens places.- The digit 5 is in the tenths place.- The digit in the ones place is 2 more than the digit in the thousandths place.What is Tyrone's number?____ ____ . ____ ____ ____ Compute: -1 2/3 * 2 1/7 NEED HELP ASAP Prompt 1: Take the position of the colonist and explain how you might feel in this situation and why?Prompt 2: Now take the position of the Native Americans and explain how you might feel in this situation and why?Prompt 3: Finally, be objective and write how you think the Native Americans and settlers could have both existed peacefully together Identify the error in the following statement: Photosynthesis is a chemical reaction that takes place in the chloroplasts of green plant cells, where chemical energy is used to convert carbon dioxide and water into glucose and oxygen.A. chloroplasts should be mitochondriaB. water and oxygen should be switchedC. No errorsD. chemical should be light Write a formula for g(x) in terms of x. Area= cm2Help me please thank you so much :) Study the poster created in California in 1876.What is the primary message of this poster?California is an excellent location for immigrants who are seeking land to farm.California has many immigrants who have grown a large supply of healthy crops.California is the best source for government loans for building railroads in the state.California has the most plentiful resources for people who want a healthy lifestyle. In a tavern, an intoxicated woman threatened to slash a man with a broken beer bottle. Another customer, who had not been threatened by the woman, forcefully grabbed the woman and locked her in the tavern's storeroom until the police would arrive. In the process, although the customer used reasonable force, the customer badly sprained the woman's wrist. Is the woman likely to recover in an action against the customer